LSAT and Law School Admissions Forum

Get expert LSAT preparation and law school admissions advice from PowerScore Test Preparation.

User avatar
 Dave Killoran
PowerScore Staff
  • PowerScore Staff
  • Posts: 5852
  • Joined: Mar 25, 2011
|
#79284
Complete Question Explanation
(The complete setup for this game can be found here: lsat/viewtopic.php?t=17034)

The correct answer choice is (C)

From our discussion of the setup, we determined that M had to be connected to one and only one other chalet (and a glance at the six solutions confirms that fact). Thus, answer choice (C) is correct.
 Agent00729
  • Posts: 29
  • Joined: Jan 25, 2021
|
#83538
i'm confused about how C is the answer. Isn't M prevented from crossing the J-N line, meaning K, L, and O (3 chalets) cannot be connected to M?

Thanks!
 Jeremy Press
PowerScore Staff
  • PowerScore Staff
  • Posts: 1000
  • Joined: Jun 12, 2017
|
#83558
Hi Agent00729,

First thing to do here is check out the templates for this game, because they really help clarify the options. Those templates are depicted here: lsat/viewtopic.php?t=17034.

Then let's dig in on the language of the question stem, which says, "Which one of the following chalets cannot be directly connected by segments of the path to exactly two other chalets?" You could read an implied phrase here: "Which one of the following chalets cannot be directly connected (in any given solution) by segments of the path to exactly two other chalets?" In other words, they're asking you to think about the individual solutions to the game, and identify a chalet that never has exactly two other direct connections in any individual solution. In the templates linked above, we can see that M is always linked to either J or N, but never to both simultaneously. If you tried to link M to exactly two other chalets (to both J and N simultaneously), you wouldn't be able to link all chalets together in the path (because you only have 5 segments to work with).

I hope this helps!
 Agent00729
  • Posts: 29
  • Joined: Jan 25, 2021
|
#83562
Makes sense, thanks a lot! Looks like I misread this one. I thought they were saying that there were exactly two other chalets on the board that M could never connect to.

Get the most out of your LSAT Prep Plus subscription.

Analyze and track your performance with our Testing and Analytics Package.